The points ( 4,0 ) and ( 7, 6) lie on a particular line What is its equation in slope-intercept form of this line?

Answers

Answer 1

Answer:

slope =2

Step-by-step explanation:

or whats your options


Related Questions

A point $P$ is randomly selected from the square region with vertices at $(\pm 2, \pm 2)$. What is the probability that $P$ is within one unit of the origin

Answers

If P = (X, Y) is a point in the given square, then X and Y are i.i.d random variables each with distribution

[tex]\displaystyle P(X = x) = \begin{cases}\dfrac14 & \text{if } -2 \le x \le 2 \\ 0 & \text{otherwise}\end{cases}[/tex]

and so the joint density of X and Y is

[tex]\displaystyle P(X = x, Y = y) = \begin{cases}\dfrac1{16} & \text{if }-2 \le x \le 2 \text{ and } -2 \le y \le 2 \\ 0 &\text{otherwise}\end{cases}[/tex]

We want to find P(X² + Y² ≤ 1). Points that satisfy this inequality lie in the set

R = {(x, y) : -1 ≤ x ≤ 1 and -√(1 - x²) ≤ y ≤ √(1 - x²)}

but we can more easily describe the region in polar coordinates by setting

x = r cos(t) and y = r sin(t)

so that the set R is identical to

R' = {(r, t) : 0 ≤ r ≤ 1 and 0 ≤ t ≤ 2π}

Integrate the joint density over R' :

[tex]\displaystyle P(X^2 + Y^2 \le 1) = \iint_R \frac1{16} \, dx \, dy[/tex]

[tex]\displaystyle P(X^2 + Y^2 \le 1) = \iint_{R'} \frac r{16} \, dr \, dt[/tex]

[tex]\displaystyle P(X^2 + Y^2 \le 1) = \int_0^{2\pi} \int_0^1 \frac r{16} \, dr \, dt[/tex]

[tex]\displaystyle P(X^2 + Y^2 \le 1) = \int_0^{2\pi} \frac{1^2 - 0^2}{32} \, dt[/tex]

[tex]\displaystyle P(X^2 + Y^2 \le 1) = \frac1{32} \int_0^{2\pi} dt[/tex]

[tex]\displaystyle P(X^2 + Y^2 \le 1) = \frac{2\pi-0}{32}[/tex]

[tex]\displaystyle P(X^2 + Y^2 \le 1) = \boxed{\frac{\pi}{16}}[/tex]

Select the correct answer.
Which expression is a factor of this polynomial?
13 + 2x^2-9x-18
O
O A. (x - 2)
B. (x + 1)
Oc. (x - 6)
D. (x + 2)

Answers

Answer:

i think i'ts D. (x +2)

Step-by-step explanation:

The slope of a line is -4, and the y-intercept is -3. What is the equation of the line written in slope-intercept form?

y = -4x - 3
y = 4x - 3
y = -4x + 3
helppppppppppppppppppppppppppppppppppppppppppppppppppppppppp

Answers

Answer:

y = -4x - 3

Hope this helps and if it did maybe mark me as brainliest : ) ?

Evaluate the expression below if a = -8, b = 17, and c = 21. a^2-(b+c)

Answers

Answer:

26

Step-by-step explanation:

a^2-(b+c)= (-8)^2-(17+21)=64-38=26

Which function has a greater rate of change?​

Answers

[tex]\huge\bold{\purple{\bold{⚡EuroNow⚡}}} [/tex]

[tex]\huge\underline\mathtt\colorbox{cyan}{Its easy}[/tex]

The function for EuroNow has a greater rate of change

What is 32/48 in simplest form?

A.2/3

B.3/4

C.4/6

Answers

Answer: A) 2/3

--------------------------------

32/48

Divide the numerator and denominator by 16:

2/3

<3

Answer:

A. 2/3

Step-by-step explanation:

32 = 2 x 16

48 = 3 x 16

32 / 48

= ( 2 x 16 ) / ( 3 x 16 )

= 2 / 3

1 7/8 divided by 2 2/5 if give good response = brainlist

Answers

Answer:

75/96 or in its simplest form, 25/32

Step-by-step explanation:

1. Convert both fractions into improper fractions

1 7/8 becomes 15/8 (1 × 8 = 8, 8 + 7 = 15)2 2/5 becomes 12/5 (2 × 5 = 10, 10 + 2 = 12)

2. Use KFC

Keep the first fraction the sameFlip the second fractionChange the sign from ÷ to ×

15/8 ÷ 12/5 = 15/8 × 5/12

15 × 5 = 758 × 12 = 9675/96 = 25/32 (divide the numerator and denomiator by 3)

Hope this help!

The position of cat and rat are shown in a graph paper.Find the coordinates of cat and rat where they lie and the distance between them.​

Answers

Cat(-5,0)Rat(5,5)

Distance:-

[tex]\\ \sf{:}\Rrightarrow \sqrt{(x_2-x_1)^2+(y_2-y_1)^2}[/tex]

[tex]\\ \sf{:}\Rrightarrow \sqrt{(5+5)^2+(5-0)^2}[/tex]

[tex]\\ \sf{:}\Rrightarrow \sqrt{10^2+5^2}[/tex]

[tex]\\ \sf{:}\Rrightarrow \sqrt{125}[/tex]

[tex]\\ \sf{:}\Rrightarrow 11.2[/tex]

What have you learned about geometric relationships?
PLEASE HELP 15 POINTS

Answers

Answer:

that you can define a tangent relationship between a line and an arc. If the adjoining elements change, the tangent relationship is maintained between the elements. Geometric relationships control how a sketch changes when edits are made.

Step-by-step explanation:

i took this i guess and it was gud

Can anyone help me with this

Answers

Answer:the 3rd one

Step-by-step explanation:

When determining domain it is important to work from

Answers

Answer:

use graphs

Step-by-step explanation:

Another way to identify the domain and range of functions is by using graphs. Because the domain refers to the set of possible input values, the domain of a graph consists of all the input values shown on the x-axis. The range is the set of possible output values, which are shown on the y-axis.

What is the slope in the picture?

Answers

Answer:

positive; sence it's starting from the bottom and going up its positive

positive!! i think it is because its going up a hill and it started at the bottom!!! hope this helps

Find the slope ... ...

Answers

Answer:

-2

Step-by-step explanation:

rise over run = 4/-2 = -2

15. Describe and correct the error in finding the intercepts of the graph of the equation.
Х
6x + 9y
= 18
6x + 9(0) = 18
6x = 18
6x + Oy 18
6(0) + 9y = 18
9y =
= 18
X = 3
y = 2
The x-intercept is at (0,3), and the y-intercept is at (2, 0).

Answers

Answer:

Wrong order!

Step-by-step explanation:

There's a reason why coordinates are called an "ordered pair", and for every point (a, b) a is the x coordinate and b is the y coordinate. The calculations are correct, the way reporting them are not: the point should be (3.0) and (0,2) - in fact, all points on the x axis are of the form (p, 0) and all points on the y axis of the form (0,q)

explain for a brainlist !!!!!!!!!!

Answers

Step-by-step explanation:

cross multiply

so it will be

2y= -1x+5

2y+x=5

thats the only part i know

Which of the following describes the transformation from Figure 1 to Figure 2? On a coordinate plane, figure A B C D E has points (negative 3, 5), (negative 2, 5), (negative 1, 4), (negative 2, 3), (negative 5, 3). Figure A prime B prime C prime D prime E prime has points (2, 2), (3, 2), (4, 1), (3, 0), (0, 0). CLEAR CHECK translation 2 units to the right and 3 units down translation 3 units to the left and 2 units up translation 5 units to the right and 3 units down translation 5 units to the left and 3 units up

Answers

Answer:

a

Step-by-step explanation:

The transformation from Figure 1 to Figure 2 is:

The transformation of 5 units to the right and 3 units down.

Option C is the correct answer.

What is translation?

It is the movement of the shape in the left, right, up, and down directions.

The translated shape will have the same shape and shape.

There is a positive value when translated to the right and up.

There is a negative value when translated to the left and down.

We have,

A B C D E has points (-3, 5), (-2, 5), (-1, 4), (-2, 3), and (-5, 3).

A' B' C' D' E' has points (2, 2), (3, 2), (4, 1), (3, 0), and (0, 0).

Now,

A = (-3 + 5, 5 - 3) to A' = (2, 2)

B = (-2 + 5, 5 - 3) to B' = (3, 2)

C = (-1 + 5, 4 - 3) to C' = (4, 1)

D = (-2 + 5, 3 - 3) to A' = (3, 0)

E = (-5 + 5, 3 - 3) to E' = (0, 0)

We see that,

There is a translation of 5 units to the right and  3 units to the down.

Thus,

The transformation of 5 units to the right and 3 units down.

Learn more about translation here:

https://brainly.com/question/12463306

#SPJ1

$4 is what percent of 50

Answers

Answer:

8%

Step-by-step explanation:

4/50 = 0.08 = 8%

Steps to solve "what percent is 4 of 50?" If you are using a calculator, simply enter 4÷50×100 which will give you 8 as the answer.

How many 2 digit numbers have unit digit 6 but are not perfect squares

Answers

9514 1404 393

Answer:

  7

Step-by-step explanation:

Of the 9 2-digit numbers ending in 6, only 2 are perfect squares: 16 and 36. The other 7 are not perfect squares.

You rent an apartment that costs $1000 per month during the first year, but the rent
is set to go up $160 per year. What would be the monthly rent during the 10th year of
living in the apartment?

Answers

Answer: In the tenth year your rent would be 2440 a month.

Hope this helps!

HELP PLEASE I CANT FIGURE THIS OUT

Answers

I don’t understand what’s the problem?

Answer:

the slope is 1/4. the equation would be y=1/4x-7 I think

Step-by-step explanation:

equation of the line that passes through the points (7, 6) and (-2, -3)?

Answers

Answer:

y=x-1

Step-by-step explanation:

Hi there!

We want to find the equation of the line that passes through the points (7, 6) and (-2, -3)

There are 3 ways to write the equation of the line:

Slope-intercept form, which is y=mx+b, where m is the slope and b is the y intercept Point-slope form, which is [tex]y-y_1=m(x-x_1)[/tex], where m is the slope and [tex](x_1, y_1)[/tex] is a pointStandard form, which is ax+by=c, where a, b, and c are free integer coefficients, but a and b cannot be 0, and a cannot be negative

The most common (and usually, the easiest way) would be slope-intercept form, so let's write it that way

First, we'll need to find the slope of the line

The slope can be found using the formula [tex]\frac{y_2-y_1}{x_2-x_1}[/tex], where [tex](x_1, y_1)[/tex] and [tex](x_2, y_2)[/tex] are points

We have two points, which is needed, but let's label their values in order to avoid any confusion:

[tex]x_1= 7\\y_1=6\\x_2=-2\\y_2=-3[/tex]

Now substitute these values into the formula to find the slope (m):

m=[tex]\frac{y_2-y_1}{x_2-x_1}[/tex]

m=[tex]\frac{-3-6}{-2-7}[/tex]

Subtract the numbers

m=[tex]\frac{-9}{-9}[/tex]

Divide

m=1

The slope of the line is 1

So far, we can write the equation of the line as this:

y=1x+b, or y=x+b

We'll need to find b

As the equation passes through both (7,6) and (-2, -3), we can use either one of them to solve for b

Taking (7, 6) for instance, substitute 7 as x and 6 as y:

6=1(7)+b

Multiply

6=7+b

Subtract 7 from both sides

-1=b

Now substitute -1 as b:

y=x-1

Hope this helps!

Perpendicular to y=-2x+1, but passes through (-4,4)

Answers

y=1/2x+6 is the new equation
Remember, the perpendicular slope is always the opposite reciprocal so the slope of this would be 1/2. Then, plug in the new slope along with the point in slope intercept form to solve for b.

A line passes through point A (12,18). A second point on the line has an x-value that is 125% of the x-value of point A and a y-value that is 75% of the y-value of point A . Use point A to write an equation of the line in point-slope form.

Answers

The answer is 18, 0

You gotta trust me

If the length of each side of a cuboid decreases by 20%, find the percentage decrease in its volume.
Solution : (Identify the ratio of the length of one edge of the original cuboid and the length of one edge of the new cuboid)​

Answers

Answer:

Step-by-step explanation:

(1+25 /100) (1-20/100) (1-50/100)  <1

5/4 x 4/5 x 1/2 <1

Decrease in volume (in percent)

(1+25 /100) (1-20/100) (1-50/100)  x 100

=48.8%

1. Mitch Hart's gross pay for this week is $425.78. He is single and claims 2 6 points

allowances. Using the percentage method of withholding, what amount

will Mitch's employer withhold from his pay for federal income tax?

Percentage Method of Withholding - Single Person

Each weekly allowance is $63.46.

Weekly Taxable The amount of income tax to

Wage

withhold is

Over But not

over

$51.00 $195.00 S0.00 plus (10% of the excess

over $51.00)

195.00 645.00 $14.40 plus (15% of the excess

over $195.00)

645.00 1482.00 $81.90 plus (25% of the excess

over $645.00)

Answers

Using the percentage method, the amount that Mitch's employer is going to withhold is $29.979

The total wage payment usually received by Mitch monthly is = $425.78Each weekly allowance is $63.46, It is noted that Mitch claims two allowances = $63.46 × 2= $126.92

Now, the amount subject to withholding from his total wage payment is;

= $(425.78 - 126.92)= $298.86

Provided that the weekly taxable wage is more than $195 but not up to $645; Then, by using the percentage method, the Federal income tax can be computed as:

= 15% of $(298.86 - 195) + $14.40= 15% of $103.86 + $14.40= $15.579 + $14.40= $29.979

Therefore, we can conclude that the amount that Mitch's employer is going to withhold is $29.979

Learn more about taxable income here:

https://brainly.com/question/17347618?referrer=searchResults

Help someone please just one and 2 that is all whoever answers 1 and 2 correctly, will get marked their answer the brainliest please​

Answers

Answer:

Step-by-step explanation:

answer please help me out

Answers

Answer:

Hey just search up an algebraic calculator I finished my homework with it lol

Step-by-step explanation:

15 points and brainliest if right!!

Answers

Answer:

y + 5x = -7

Step-by-step explanation:

Hello there?

y + 3 = -5(x + 2)

LHS

Open the brackets

= -5x - 10

RHS remains

Combine the two:

y + 3 = -5 - 10

Collecting the like terms and taking the value of x to the LHS

=> y + 5x = - 7

I hope this helps. Have a nice studies

6. What is an equation in slope-intercept form for the line that passes through the points and (3, 1)? y= 3x + 1 y=x-3 y=2x + 5 y= 2x - 5​

Answers

Answer:

The correct answer is D) y = 2x - 5

To find the equation of the line, start by finding the slope. You can do this by using the slope formula below.

m(slope) = (y2 - y1)/(x2 - x1)

m = (1 - -3)/(3 - 1)

m = 4/2

m = 2

Now that we have the slope, we can use it along with either point in point-slope form to get the equation.

y - y1 = m(x - x1)

y - 1 = 2(x - 3)

y - 1 = 2x - 6

y = 2x - 5

Answer: y=2x-5

the points are 4 points away from each other vertically, and 2 away horizontally. This makes the slope 4/2 or just 2.

take the slope and go backwards until you hit the y axis and the y-intercept is -5

this means the equation is y=2x-5

There are 5 boxes and each box weighs x pounds. Write the expression for the total weight of the
boxes.

Answers

Answer:

5x pounds

Step-by-step explanation:

Here's the solution if you need, hope it's help

Other Questions
Which word does not belong?un restaurantO un supermarcho une banqueun march Which of the following statements is TRUE during depolarization? Question 10 of 10Which of the following is the solution to the inequality below?-2/6(4-x) HIIIIIIIIIIIIIIIIIIIIIIIIIIIIIIIIIIII x=4y-72x+9y=-31 What be the answer These trianglesare congruent bythe trianglecongruencepostulate [? ].A. ASAB. Neither, they are not congruentC. AAS What mass of LiOH would need to be dissolved in 500.0 mL of water to produce a solution with a pH of 12.40 sure to show all work.1. You invest $1,000 with an interest rate of 4% annually (annually means once each year). How muchinterest will you earn after one year?interest Question 3 of 20When you practice assertive communication, you are trying to persuadeothers to agree with your point of view.O A. TrueB. False There is a line that includes the point(10, 5)and has a slope of 1 Whatis its equation in slope-intercept form?So far I have y=1x+what's the y? What problems did Elizabeth I face in her own country Why is it possible to collect hydrogen gas over water? From first principles, find the indicated derivatives What is 2x-3y=12 on a graph? What was the big bang? if a/3=b/2, what are the following ratios?a+b/b A bike take 56 minutes to travel 8 km? How many minutes will take to travel 15? what are the examples of syneedoche from chapter lastlesson class 12 What selfie captions can you use with the words Adjacent, abundant, ancient, deterrent, dependent, extravagant, imminent, incident, restaurant, tyrant Riddle me this: You're always right, what is your sine?Answer: #1What does that even mean??